logarithmic differentiation for

[tex]y = x {}^{2} [/tex]

someone help me

Answers

Answer 1

Answer:

[tex]\boxed {\frac{dy}{dx}= 2x}[/tex]

Step-by-step explanation:

Solving :

⇒ log y = log (x²)

⇒ log y = 2 log x

⇒ [tex]\mathsf {\frac{1}{y} \frac{dy}{dx} = \frac{1}{x} \times 2}[/tex]

⇒ [tex]\mathsf {\frac{dy}{dx}= 2x}[/tex]

Answer 2

Answer:

y’ = 2x

Step-by-step explanation:

Let y = f (x), take the natural logarithm of both sides ln (y) = ln (f (x))

ln (y) = ln (x²)

Differentiate the expression using the chain rule, keeping in mind that y is a function of x.

Differentiate the left hand side ln (y) using the chain rule.

y’/y = 2 In (x)

Differentiate the right hand side.

Differentiate 2 ln (x)

y’/y = d/dx = [ 2 In (x) ]

Since 2 is constant with respect to xx, the derivative of 2 ln (x) with respect to x is 2 d/dx [ln (x)]

y’/y = 2 d/dx [In (x)]

The derivative of ln (x) with respect to x is 1/x.

y’/y = 2 1/x

Combine 2 and 1/x

y’/y = 2/x

Isolate y' and substitute the original function for y in the right hand side.

y’ = [tex]\frac{2}{x}[/tex] x²

Factor x out of x².

y’ = [tex]\frac{2}{x}[/tex] (x * x)

Cancel the common factor.

y’ = [tex]\frac{2}{x}[/tex] (x * x)                     (The x that is under 2 and the other x that I have underlined are the ones that cancel out)  

Rewrite the expression.

y’ = 2x

So therefore, the answer would be 2x.


Related Questions

Could someone explain why the value of x is (-2).​

Answers

Answer:

x = -2

Step-by-step explanation:

2x^3 +16 = 0

We are solving for x

Subtract 16 from each side

2x^3 +16- 16 = 0-16

2x^3 = -16

Divide each side by 2

(2x^3 ) /2 = -16/2

x^3 = -8

Take the cube root of each side

[tex]\sqrt[3]{x^3} = \sqrt[3]{-8}[/tex]

x = -2

Answer:

[tex]x=-2[/tex]

Step-by-step explanation:

Given equation:

[tex]2x^3+16=0[/tex]

To solve for the unknown variable x, apply arithmetic operations to isolate the variable.

Subtract 16 from both sides:

[tex]\implies 2x^3+16-16=0-16[/tex]

[tex]\implies 2x^3=-16[/tex]

Divide both sides by 2:

[tex]\implies \dfrac{2x^3}{2}=\dfrac{-16}{2}[/tex]

[tex]\implies x^3=-8[/tex]

Take the cube root of both sides:

[tex]\implies \sqrt[3]{x^3}=\sqrt[3]{-8}[/tex]

[tex]\implies x=-2[/tex]

When taking the cube root of a negative number, the result will be negative.  To understand why, examine what happens when we cube a negative number.

When a number is cubed, it is multiplied by itself, then by itself again.

When multiplying a negative number by another negative number, the result is always positive.

When multiplying a positive number by a negative number, the result is always negative.

Therefore:

[tex]\begin{aligned}\implies (-2)^3 &= -2 \cdot -2 \cdot -2\\ & = 4 \cdot -2\\& = -8\end{aligned}[/tex]

So if -8 is cube rooted, the result is -2.

What is the solution of the inequality sh
below?
9+b≤2

Answers

Answer:

b≤-7

Step-by-step explanation:

b+9≤2

Subtract 9 from both sides:

b+9-9≤2-9b≤-7

9+b≤2

subtract 9 on each side which will be -7

the answer will be b≤-7

A. Yes, the points shown on the line would be part of y=0.5x
B. Yes, all proportions can be shown on a graph of this line
C. No, the points shown would not be part of y=0.5x
D. No, the proportions cannot be represented on a graph

Answers

Answer:

B

Step-by-step explanation:

given the graph of y = 0.5x

Then any point on the line satisfies y = 0.5x

consider the points given

x = 1 then y = 0.5 × 1 = 0.5 ⇒ (1, 0.5 ) is on the line

x = 4 then y = 0.5 × 4 = 2 ⇒ (4, 2 ) is on the line

Which expression has a value of 16 when n = 5?
StartFraction 25 Over n EndFraction + 7
30 minus 3 n
7 + StartFraction 45 Over n EndFraction
n cubed minus 114

Answers

Answer:

7+45/n or 5

Step-by-step explanation:

45 divided by n or technically 5 is 9 plus 7 is 16 :)

Answer:7+45/n or 5

Step-by-step explanation:

What expression below is equivalent to -71n -
29?
A) -29 - 71n
B) 71n + (-29)
C) 29 + 71n
D) -29 - (-71n)

Answers

Answer:

A

Step-by-step explanation:

both numbers are negative, you can rearrange them in any order provided you keep the sign

Answer: Choice A) -29 - 71n

Reason:

An expression in the form -a-b is the same as -a + (-b)

Recall we can add two numbers in any order we want. For instance, 2+3 = 3+2 since both sides are equal to 5. This is the commutative property of addition.

Therefore, the -a + (-b) is the same as -b + (-a) which in turn becomes -b - a

In short:  -a - b = -b - a

This rule lets us say that -71n - 29 = -29 - 71n

\In a mouse population, some mice have thicker fur than others, but there are more mice with thinner fur. The climate in which these mice live has slowly gotten colder.

How will this change in the environment most likely affect the mouse population?

The adaptation of thinner fur will be selected over thicker fur, and the population will remain unchanged.
The adaptation of thicker fur will be selected over thinner fur, and the population will remain unchanged.
The adaptation of thinner fur will be selected over thicker fur, and the population will evolve.
The adaptation of thicker fur will be selected over thinner fur, and the population will evolve.

Answers

The adjustment of thicker fur will be selected over thinner fur, and the population will develop. Then the correct option is D.

What is decision-making?

Determining the proper option, acquiring evidence, and exploring various options are all steps in the decision-making process.

In a mouse population, some mice have thicker fur than others, but there are more mice with thinner fur.

The climate in which these mice live has slowly gotten colder.

The change in the environment most likely affect the mouse population will be

The adjustment of thicker fur will be selected over thinner fur, and the population will develop.

Then the correct option is D.

More about the decision-making link is given below.

https://brainly.com/question/3369578

#SPJ1

Marcela was assigned some math problems for
homework. She answered half of them in study hall.
After school she completed seven more. If she still
has 11 problems to do, how many problems were
assigned?

Answers

Answer:

36 problems

Step-by-step explanation:

so let's say x = how many problems Marcela was assigned

she completed half of them so the amount of problems left is x/2

She then completed 7 more so now we can subtract 7. We then set this to 11 to solve for x

x/2 - 7 = 11

x/2 = 18

x = 36

The steps to simplify the expression -100 ÷ 3 x (-0.6) are shown.

Answers

Answer:

That is the answer for you step 2

The top of a tree makes angles s and t with points K and L on the ground, respectively, such that the angles are complementary. Point K is x meters and point L is y meters from the base of the tree.

1. In terms of x and y, find the height of the tree. Include your calculations.
2. If m∠t = 40° and y = 8 meters, calculate the height of the tree, rounded to two decimal places.

Answers

1. In terms of x and y, the height of the tree is [tex]h = y \times tan \ t^\circ[/tex] and[tex]h = x \times tan \ s^\circ[/tex]

2. The height of the tree is 6.71 meters

Trigonometry

From the question, we are to determine the height of the tree in terms of x and y

Using SOH CAH TOA, we can write that

[tex]tan \ t^\circ =\frac{h}{y}[/tex]

∴ [tex]h = y \times tan \ t^\circ[/tex]

Also,

[tex]tan \ s^\circ =\frac{h}{x}[/tex]

∴ [tex]h = x \times tan \ s^\circ[/tex]

2. If m∠t = 40° and y = 8 meters,

Then, the height of the tree

h = 8 × tan40°

h = 6.71 meters

Hence, the height of the tree is 6.71 meters

Lear more on Trigonometry here: https://brainly.com/question/15977788

#SPJ1

Genesis is going to see a movie and is taking her 3 kids. Each movie ticket costs $15 and there are an assortment of snacks available to purchase for $3 each. How much total money would Genesis have to pay for her family if she were to buy 6 snacks for everybody to share? How much would Genesis have to pay if she bought x snacks for everybody to share?

Total cost with 6 snacks:
Total cost with x snacks:.
need answer quickly as possibly!

Answers

Answer #1:

The total cost with 6 snacks is $78.

Answer #2:

The total cost with x snacks is represented by the expression 60 + 3x.

Step-by-step explanation for answer #1:

Starting off with what we know:

Genesis has 3 kids, so there are 4 people total including Genesis.Each movie ticket is $15.Each snack is $3.

To answer the question of how much money Genesis would have to pay for her family if she bought 6 snacks for everybody, first consider the total cost of the movie tickets.

Multiply the total number of people by the cost of each movie ticket:

[tex]4* 15=60[/tex]

Genesis will be paying $60 total only to see the movie with her 3 kids. To find the total cost of both seeing the movie and buying 6 snacks, simply multiply the number of snacks by the cost of each snack:

[tex]6 * 3=18[/tex]

Then, add the total cost of the movie tickets by the total cost of the snacks to achieve your first answer:

[tex]60+18=78[/tex]

The total cost with 6 snacks is $78.

Step-by-step explanation for answer #2:

To find the total cost of "x" snacks (x is being used as a variable term to represent a quantity subject to change), we can create an algebraic expression.

Let "t" represent the total cost.

Since we've established that finding the total cost is just a matter of adding the total cost of movie tickets (60) and the total cost of snacks (3 multiplied by the number of snacks), let "x" represent the number of snacks in the equation to find the total cost with x snacks:

[tex]60+3x=t[/tex]

The total cost with x snacks is represented by the expression 60 + 3x.

Based on the given data, the total cost of 6 snacks: $78. total cost with x snacks: $60 + (3x).

Use the concept of multiplication defined as:

To multiply means to add a number to itself a particular number of times. Multiplication can be viewed as a process of repeated addition.

Given that,

Genesis is going to see a movie with her 3 kids.

Each movie ticket costs $15.

There are snacks available for purchase at $3 each.

Genesis wants to buy 6 snacks for everybody to share.

To calculate the total cost for Genesis and her 3 kids,

Consider the movie tickets and the snacks.

The cost of the movie tickets can be calculated by multiplying the number of people (4 in this case) by the cost of each ticket ($15).

So, Genesis would have to pay 4 x $15 = $60 for the movie tickets.

Now, let's calculate the total cost if Genesis were to buy 6 snacks for everybody to share.

Each snack costs $3, so 6 snacks would cost 6 x $3 = $18.

Therefore, the total cost for Genesis would be,

$60 (movie tickets) + $18 (snacks) = $78.

If we consider x snacks for everybody to share,

The total cost for snacks would be x $3.

So, the final cost would be $60 (movie tickets) + (3x) (snacks).

Learn more about the multiplication visit:

https://brainly.com/question/10873737

#SPJ3

Two gymnasts are running toward each other in a floor routine, and they plan to precisely time a flip to stay synchronized for the audience. The path of the gymnasts is parabolic and modeled by the following equations, where y is the height of the flip and x is the time in seconds:

Answers

Answer:

4 secs

Step-by-step explanation:

Since they have to be synchronized, their parabolic equations must be equal to one another

3(t^2 + 9 - 6t) - (t^2 + 25 - 10t) - t + 2 = 0

2t^2 - 9t + 4

(2t - 1)(t - 4) = 0

t = 1/2, 4

From the options, the answer is 4

The gymnasts will be at the same height during their flips at two different times: 1/2 seconds and 4 seconds.

so, correct option is: C.

Here, we have,

To determine when the gymnasts will be at the same height during their flips, we need to find the time (x) at which the equations for y are equal.

The given equations are:

y = –(x – 5)² + 3

y = –3(x – 3)² + x + 1

Setting the two equations equal, we have:

–(x – 5)² + 3 = –3(x – 3)² + x + 1

Expanding the squared terms:

–(x² – 10x + 25) + 3 = –3(x² – 6x + 9) + x + 1

Simplifying the equation:

–x² + 10x – 25 + 3 = –3x² + 18x – 27 + x + 1

Combining like terms:

–x² + 10x – 22 = –3x² + 19x – 26

Rearranging the equation:

2x² - 9x + 4 = 0

To solve this quadratic equation, we can factor it:

(2x - 1)(x - 4) = 0

Setting each factor equal to zero:

2x - 1 = 0 or x - 4 = 0

Solving for x:

2x = 1 or x = 4

Dividing by 2:

x = 1/2 or x = 4

Therefore, the gymnasts will be at the same height during their flips at two different times: 1/2 seconds and 4 seconds.

To learn more on equation click:

https://brainly.com/question/14468218

#SPJ2

2. The smallest number from the given numbers 6895, 4875 and 5689 will have its smallest digits at place. (A) ​

Answers

The smallest digit of the smallest number is at the thousands place.

What is Number System ?

The system of representing numbers is called Number System.

The given numbers are 6895 , 4875 , 5689

The smallest number among these is 4875

The smallest digit in this number is 4

4 is at thousands place

Therefore , The smallest digit of the smallest number is at the thousands place.

To know more about Number System

https://brainly.com/question/22046046

#SPJ1

write the faction in the simplest form 28/36

Answers

Answer:

7/9

Step-by-step explanation:

Divide the numerator and denominator by 4.

28/36 = 7/9

Answer:

7/9

Step-by-step explanation:

28 and 36 are both divisible by 4.

28÷4 is 7 and

36÷4 is 9

28/36 = 7/9

You can walk over the mountains to
your campsite or take a safer, easier trip around the mountains. You
are on the ground at P(0, 0) and the campsite is at C(-2, 7). You
travel around the mountain by going to A(2, 5). The coordinate
system is measured in meters. Draw a diagram of the situation.
Find PA and AC.

Answers

Answer:

PA = [tex]\sqrt{29}[/tex]

AC =[tex]\sqrt[2]{5}[/tex]

Step-by-step explanation:

distance formula : if [tex](x_{1},y_{1}) and (x_{2},y_{2})[/tex] are two points on a line segment then distance between both the points is given by distance formula.

[tex]d = \sqrt{[(x_{2} - x_{1} )^2 +(y_{2} - y_{1})^2]}[/tex]

therefore ,

given points are , A(2, 5),  P(0, 0), C(-2, 7).

so PA = [tex]\sqrt{[({2} - {0} )^2 +({5} - {0})^2]} = \sqrt{({2} )^2 +({5} )^2}[/tex]

    PA = [tex]\sqrt{\229}[/tex]

similarly,

AC = [tex]\sqrt{[({-2} - {2} )^2 +({7} - {5})^2]} = \sqrt{({4} )^2 +({2} )^2}[/tex]

AC = [tex]\sqrt{20} = \sqrt[2]{5}[/tex]

more questions on lines and distance formula at

https://brainly.com/question/25841655

#SPJ10

perpendicular to the line y =6 - 3x; passes through (-3, -1)

Answers

Answer:

[tex]y=\frac{1}{3}x[/tex]

Step-by-step explanation:

Perpendicular lines

Perpendicular lines have slopes whose product is -1.  In other words, to find a line that is perpendicular to a given line, the new line must have a that is the opposite reciprocal of the original line.

Passing through a point
Additionally, if a line contains a point, then the point must be a solution to the equation (meaning, when you plug in the "x" and "y", the equation must be true).

Finding our perpendicular line

Work on slope first

To find the slope of the original line, rewrite the original line in slope-intercept form: [tex]y=mx+b[/tex]

[tex]y=6-3x\\y=6+(-3x)\\y=(-3x)+6\\y=-3x+6\\y=-\frac{3}{1}x+6[/tex]

In this form, the slope is the number multiplied to the "x", so the original slope is -3/1.

Thus, the slope of the new perpendicular line will be the opposite reciprocal of -3/1 ... which is 1/3.

Writing what we do know about the new perpendicular line, we have [tex]y=\frac{1}{3}x+b[/tex]

Making sure it passes through the point
This new line is also supposed to contain the point (-3,-1), so (-3,-1) must be a solution to the equation, for whatever constant-value the "b" is.  To find out the "b", substitute the known quantities, and solve:

[tex]-1=\frac{1}{3} (-3)+b\\-1=-1+b\\(-1)+1=(-1+b)+1\\0=b[/tex]

Substituting this into our equation:

[tex]y=\frac{1}{3}x+(0)\\y=\frac{1}{3}x[/tex]

Conclusion

So, the equation for a line that is perpendicular to the original line [tex]y=6-3x[/tex], and that also passes through (-3,-1) is [tex]y=\frac{1}{3}x[/tex]

Solve for x.

log x = 3

Answers

-------------------------------------------------------------------------------------------------------------

Answer:  [tex]\textsf{x = 1000}[/tex]

-------------------------------------------------------------------------------------------------------------

Given: [tex]\textsf{log(x) = 3}[/tex]

Find: [tex]\textsf{The value of x}[/tex]

Solution:  In order to solve for x we need to get rid of the log which means that we need to put it to the power of the same number as the base.  When a base isn't displayed it defaults to 10 so we raise everything to the base of 10.

Get rid of log

[tex]10^{\textsf{log}_{10}\textsf{(x)}}} = 10^{\textsf{3}}[/tex][tex]x = 10^{\textsf{3}}[/tex][tex]x = 1000[/tex]

Therefore, the final answer would be that x is equal to 1000.

Gerald purchased a rectangular plot of land. the length of the plot is 20 feet more than the width. the cost of the land was $12 per square foot. gerald also had a fence put around the entire perimeter of the plot, at a cost of s8 per linear foot. the total amount he spent on both the land and the fence was $10,560. part a write an equation in two variables for the perimeter, p, and an equation in two variables for the area, a, of the plot of land where x is the width of the plot in feet. provide evidence to support your equations

Answers

Answer:

P =2(x+ y) [tex]ft[/tex].

A =xy [tex]ft^{2}[/tex]

Step-by-step explanation:

perimeter of rectangle (P)= 2(L+B

Area of rectangle (A)        =    L×B

Given,

width = 'x'

length = y = x+20

therefore using above formula

P = 2(x+20 + x) = 2(2x+20)

A = (x+20)(x)

cost of land per square foot = $12

cost of land per linear foot   = $8

total cost of land                   = $10560

now,

P×8 + A×12   = 10560

2(2x+20)×8 +  (x+20)(x)×12 = 10560

32x + 320 + 12x²+ 240x     = 10560

12x² + 272x                         = 10240

12x² + 272x - 10240           = 0

solving this quadratic equation using Discriminant method

x = [-b ±√(b² - 4ac)]/2

where b is the coefficient of x

a is the coefficient of x²

and c is the constant term

by replacing values in the above formula

x = 19.5  , x = -42.24

as the area cannot be negative therefore -42.24 is neglected

hence x = 19.5ft

more on quadratic equations on

https://brainly.com/question/2925460

#SPJ10

In the triangle above, what is the measure of A.
M B.
M C.
M D
, cannot be determined

Answers

B. 45. The two angles B and A are both 45 to make 90 and angle C is a 90 degree angle so they come together for 180 degrees total

Use the drawing tool(s) to form the correct answer on the provided graph.
Graph the solution to the following linear inequality in the coordinate plane.
5x - y > -3
27
Drawing Tools
Select
Line
Dashed Line
Shaded Region
Click on a tool to begin drawing.
10
8
6

Answers

The solution to the inequality 5x - y > -3 is shown in the graph.

What is an equation?

An equation is an expression that shows the relationship between two or more numbers and variables.

An inequality shows the non equal comparison of two or more variables and numbers.

The solution to the inequality 5x - y > -3 is shown in the graph.

Find out more on equation at: https://brainly.com/question/2972832

#SPJ1

Someone please help with this!!

Answers

Answer:

A

Step-by-step explanation:

Comment (and answer)

Both start with the plus x axis as one of the arms. The other arm of 110o goes anti clockwise until it is 30 degrees into the second quadrant.

The answer can be found by using 110 - 360 = negative angle.

So 110 - 360 = - 250 which goes clockwise from the plus x axis. The answer must be A.

Make sure you understand the concept of clockwise and anti clockwise. If you don't know, ask your teacher, write it down, and try not to forget it. The distinction comes up a lot.

Which of the following are solutions to the equation below?
Check all that apply.
(3x - 5)^2 = 19

Answers

Answer:

To solve the answer, you can do it in 2 ways:

1) you could take the square root on both sides and solve for x

OR

2)you could multiply 3x-5 times itself and subtract 19 on both sides and use the grouping or quadratic formula method to find x.

Hope it helps.

Take the square root of each side of the equation to set up the solution for

Remove the perfect root factor

under the radical to solve

The complete solution is the result of both the positive and negative portions of the solution.

The result can be shown in multiple forms.

The graph of the function f(x) = –(x + 3)(x – 1) is shown below.

On a coordinate plane, a parabola opens down. It goes through (negative 3, 0), has a vertex at (negative 1, 4), and goes through (1, 0).

Which statement about the function is true?

The function is positive for all real values of x where
x < –1.
The function is negative for all real values of x where
x < –3 and where x > 1.
The function is positive for all real values of x where
x > 0.
The function is negative for all real values of x where
x < –3 or x > –1.

Answers

Answer:

The second statement is true.

Step-by-step explanation:

Draw the corresponding parabola and use it to verify  all statements.

Answer:

The function is negative for all real values of x where

x < –3 and where x > 1.

Step-by-step explanation:

The answer above is correct.

what is
f(-1)=-3x^{2}+7x+4

Answers

Step-by-step explanation:

please mark me as brainlest

f(-1) = -3(-1)^2 + 7(-1) + 4 = -3 + -7 + 4 = -6

Fill in the table with the appropriate value to describe the amount of hot dogs eaten at a hot dog eating contest.

Which value should be placed in the empty space in the table?

A. 1/6
B. 1/4
C. 1/12
D. 1/8

Answers

20 hot dogs was eaten by the 10 people who attended the hot dog eating contest.

What is an equation?

An equation is an expression that shows the relationship between two or more numbers and variables.

Let us assume that there were 10 people at the hot dog eating contest. If each person eats 2 hot dog, hence:

Number of hot dogs eaten = 2 hot dog per person * 10 people = 20 hot dogs.

20 hot dogs was eaten by the 10 people who attended the hot dog eating contest.

Find out more on equation at: https://brainly.com/question/2972832

#SPJ1

The length of a rectangle is 7 inches and the width is 42 inches. What is the ratio,
using whole numbers, of the length to the width?

Answers

Answer:

6 : 1

Step-by-step explanation:

The length is the longer side compared to the width.

Hence, the length is 42 inches and width is 7 inches.

Taking the ratio :

Length : Width42 : 77 x 6 : 76 : 1

Answer:

1 : 6

Step-by-step explanation:

42 /7 = 6

We will have to simply the ratios in order to get the answer.

area = ft
help me please ty:)​

Answers

find the length using the Pythagorean theorem.

Length = sqrt(20^2 - 12^2)

Length = 16 feet


area = 12 x 16 = 192 square feet

Find (f • g) (x) Assume x>0

Answers

Answer:

[tex]\textsf{B.} \quad (f \cdot g)(x)=10x[/tex]

Step-by-step explanation:

Given:

[tex]\begin{cases}f(x)=\sqrt{50x}\\g(x)=\sqrt{2x}\end{cases}[/tex]

[tex]\begin{aligned}\textsf{As }(f \cdot g)(x) & = f(x) \cdot g(x)\\\implies (f \cdot g)(x)& = \sqrt{50x} \cdot \sqrt{2x}\end{aligned}[/tex]

[tex]\textsf{Apply radical rule} \quad \sqrt{a}\sqrt{b}=\sqrt{ab}:[/tex]

[tex]\begin{aligned}\implies (f \cdot g)(x) &= \sqrt{50x2x}\\& = \sqrt{100x^2}\end{aligned}[/tex]

Rewrite 100 as 10²:

[tex]\implies (f \cdot g)(x)=\sqrt{10^2x^2}[/tex]

[tex]\textsf{Apply exponent rule} \quad a^bc^b=(ac)^b:[/tex]

[tex]\implies (f \cdot g)(x)= \sqrt{(10x)^2}[/tex]

[tex]\textsf{Apply radical rule} \quad \sqrt{a^2}=a, \quad a \geq 0:[/tex]

[tex]\implies (f \cdot g)(x)=10x[/tex]

Is in between 2 number on the number line

Answers

Answer:

3 and 4

Step-by-step explanation:

well you can approximate by squaring values

2^2 = 4

3^2 = 9

4^2 = 16

the value should fall somewhere between 3 and 4 since the square root of 9 is 3 and the square root of 16 is 4

simplify (3+√5)(3+√2)

Answers

When we simplify (3 + √5)(3 + √2), the result obtained is:

9 + 3√2 + 3√5 + √10

Surd operation

a√b × c√d = (a × c)√(b × d)

How to simplify (3 + √5)(3 + √2)

(3 + √5)(3 + √2)

Expand by clearing the bracket

3(3 + √2) + √5(3 + √2)

9 + 3√2 + 3√5 + √10

Thus,

(3 + √5)(3 + √2) = 9 + 3√2 + 3√5 + √10

Learn more about surd:

https://brainly.com/question/24700530

#SPJ1

Mr wong predicted that he would sell 45 automobiles , he actually sold 48

Answers

Answer:

well done mr wong lol

Step-by-step explanation:

what's the question is it the percent he beat his prediction?

if it is then its 48 divided by 45 times 100

this equals 106.67%

then take this away from his prediction percent originally which is 100%

106.67-100= 6.67%

Other Questions
a ball is thrown upward from the ground with an initial velocity of 50m/s. what is the total time spent by the ball in the air (g=10m/s2) Under her cell phone plan, Sarah pays a flat cost of $69 per month and $4 per gigabyte. She wants to keep her bill at $83.80 per month. Write and solve an equation that can be used to determine g, the number of gigabytes of data Sarah can use while staying within her budget. The equation for the pH of a substance is pH = -log[H*], where H+ is the concentration of hydrogen ions. A basicsolution has a pH of 11.2. An acidic solution has a pH of 2.4. What is the approximate difference in the concentraticof hydrogen ions between the two solutions?1.6x10-9O4.0x10-3O 6.7x10-1O 1.6x1011 How are manures different from fertilizers? List out the impacts of using fertilizers over and over and how can it be prevented. Jamaal knows that it is certain that he will win the election because he is the only person who is running for class treasurer. which value represents the probability that he will win the election? 0 one-fourth three-fourths 1 Analyze the diagram below and complete the instructions. Find the value of x and the value of y The statement a > 2 and a < 5 is true when a is equal to A. 5 B. 3 C. 10 D. 2 Which of the following terms best describes a vertical line that the graph of a function approaches but never intersects Please help me outIt would be appreciated What is the role of a catalyst in a chemical reaction?A increase the concentration of reactantsB increase the rate of the reactionC increase the temperature of a reactionD increase the pressure applied to the reaction Two measures of two supplementary angles are in the ratio of 2.3 find the measurments of the two angles. The following are scores for students in Ms. Kennedy's math class.What is the range of the scores?451560600422603514A 109B 149C 178D181 The number of young drivers involved in motor vehiclecrashes each year has declined substantially in recentyears, for teenage drivers of all ages and in crashes of allseverities. The number of people injured annually incrashes involving teen drivers declined by 51% between1994 and 2013, and the number killed each year in teendriver crashes declined by 56%. Most of the decline ininjuries and virtually the entire decline in fatalitiesoccurred between 2004 and 2013. While the reasons forthe declines are not entirely clear, numerous studies haveshown that strong state GDL systems have contributedsubstantially to reductions in injury and fatal crashes ofdrivers aged 15-17. Research also suggests that economicfactors including rising gas prices and the economicrecession of 2008 resulted in substantial declines in teendriving and thus teen crashes as well.Intro-Teen Driver Crashes: 1994-2013,AAA Foundation for Traffic SafetyHow could the reliability of this source best bedescribed?The source is reliable because it is a primarysource.O The source is not reliable because the author isanonymous.The source is not reliable because the authorseems to be biased.O The source is reliable because it provides facts anddetails by an expert.Done 1. When building a house using bricks a damp course is laid just above the brick foundation. Explain why the damp course is necessary 20. Which group of people is better atunderstanding their own emotions and theemotions of others, which helps them to bebetter communicators?A. People who don't understandpersonality typesO B. People with higher intelligence quotients(IQ)O C. People with higher emotionalintelligence (EQ)O D. People with introverted personalities What is another way to writeMP why prithvi Narayan Shah appointed the person favoured by the people as kaji ? explain give long answer for this question Point A is located at (2, 2), and point M is located at (1, 0). If point M is the midpoint of segment AB, find the location of point B. (0.5, 1) (4, 2) (5, 4) (1, 1) Your goals tend to set the tone for your investment program, and they play a major role in determining how conservative or aggressive you're likely to be in making investment decisions. Group of answer choices True False What important cellular process produces water molecules as it breaks down glucose in the presence of oxygen